Two 3.09 cm by 3.09 cm plates that form a parallel-plate capacitor are charged to +/- 0.617 nC. What is the electric field strength inside the capacitor if the spacing between the plates is 1.784 mm?

Answers

Answer 1

Given:

The charge on the capacitor is Q = 0.617 nC

The distance between plates is d =1.784 mm

The area is

[tex]\begin{gathered} A=\text{ 3.09}\times3.09 \\ =9.5481\text{ cm}^2 \end{gathered}[/tex]

To find the electric field strength.

Explanation:

The electric field strength can be calculated by the formula

[tex]E=\frac{Q}{\epsilon_oA}[/tex]

The constant is

[tex]\epsilon_o=8.85\times10^{-12}\text{ C}^2\text{ / N m}^2[/tex]

On substituting the values, the electric field strength will be

[tex]undefined[/tex]


Related Questions

What is the speed at the bottom of the funnel?

Answers

We will have the following:

[tex]\begin{cases}I=r^2m \\ \\ L=I\omega \\ \\ \omega=v/r\end{cases}[/tex]

So:

We will find the data we can with the given information:

[tex]\omega_1=\frac{0.3m/s}{0.2m}\Rightarrow\omega=\frac{1.5}{s}[/tex]

Also:

[tex]I\omega_1=I\omega_2\Rightarrow\omega_1=\omega_2[/tex]

So:

[tex]\frac{1.5}{s}=\frac{v}{0.03m}\Rightarrow v=0.045m/s[/tex]

So, the velocity at the bottom of the funnel is 0.045m/s.

Artificially produced radioactive isotopes are used in medicine and medical research.Question 1 options:TrueFalse

Answers

Artificial isotopes are usually absent by nature because of their unstables nature . The Isotopes usually transit into a stable Isotopes. This Isotopes are produced artificially by bombarding an atom.

Artificially radioactive Isotopes produced from stable Isotopes are used in medicine and medical research. Therefore, the answer is True.

A current of 17.0A is maintained in a single circular loop of 2.00m circumference. A magnetic field of 0.800T is directed parallel to the plane of the loop. (a) Calculate the magnetic moment of the loop. [5] (b) What is the magnitude of the torque exerted on the loop by the magnetic field?

Answers

A. The moment of the loop is 5.4 x [tex]10^{-3} Am^{2}[/tex]

B.  The torque exerted on the loop by the magnetic field is 4.32 [tex]10^{-3}[/tex] Nm.

Given that the current I in the circular loop is 17.0 mA and the circumference of the loop is 2.00 m.

The radius of the circular loop is calculated as given below.

2πr = 2

r = 2/2 x 3.14

r = 0.18m

The moment in the circular loop is calculated as given below.

M = in

M = 17 x [tex]10^{-3}[/tex] x 3.14 x 0.318²

M = 5.4 x [tex]10^{-3} Am^{2}[/tex].

The torque exerted on the loop by the magnetic field is given below:

T = MB

T = 5.4 x [tex]10^{-3}[/tex] x 0.800

T = 4.32 x [tex]10^{-3} Nm[/tex].

This is the torque of the current-carrying loop in a uniform magnetic field. This formula can be shown to be valid for loops of any shape. The loop carries a current I, has N turns, and the plane A and the normal to the loop each make an angle θ with the field B. The net force on the loop is zero. A magnetic field exerts a force on a straight wire through which an electric current flows. Apply torque to the loop of the wire carrying the current. A torque rotates an object around a fixed axis.

Learn more about The torque here:-https://brainly.com/question/20691242

#SPJ1

Margie, who weighs 624 N, is standing on a bathroom scale that weighs 51.0 N.
With what magnitude force does the floor push up on the scale?

Answers

Answer:

675N

Explanation:

624 + 51 = 675N

as the floor exerts and an equal and opposite force.

What does the top pressure gauge in (Figure 1) read? Express your answer using two significant figures.

Answers

Digital manometers can be used to measure liquids, gases, and air, depending on the device. If the instruments have an internal memory, it works quite well.

What does the top pressure gauge indicate?

Pressure Gauge in Psig Pressure as a percentage of the surrounding atmospheric pressure. The gauge pressure for this kind of pressure includes the pressure from the weight of the atmosphere because it is zero-referenced against atmospheric pressure.

These gauges typically have a measurement range of 0… 1 mbar to 0… 600 mbar and an accuracy class of 0.1 to 2.5. Although pressure gauges typically require very minimal upkeep, problems can happen.

Therefore, This tool can be used to measure pressure in a variety of locations. Measurements of negative pressure, overpressure, and differential pressure are all possible.

Learn more about pressure gauge here:

https://brainly.com/question/29341536

#SPJ2

After being pushed away from the departure gate, an aircraft is standing still with the brakes applied and the engine running on very low. When the pilot releases the brakes he finds that the plane still does not start to move. Here are three explanations for why the plane does not move. Which one do you agree with?

Answers

Since he pilot releases the brakes and he finds that the plane still does not start to move, the  explanation I agree with is option C:

When the pilot releases the brakes two forces act on it. The engines exert a force in one direction and the ground exerts a force on the wheels in the opposite direction. The plane does not move because the force exerted by the ground on the wheels is the same strength as the force of the engines.

How do planes fly and stay in the air?

The lift of the airplane is provided by the flow of air around the wings. Additionally, the wings' form contributes to lift. The airplane is being pulled toward Earth by weight. The weight of an airplane is distributed evenly from front to back during construction.

Note that the design of an airplane wing is intended to speed up airflow across the top of the wing. Air pressure decreases when it moves more quickly. Therefore, the pressure at the top of the wing is lower than that at the bottom. The pressure differential exerts a force on the wing, lifting it into the air.

Learn more about Airplanes from

https://brainly.com/question/928514
#SPJ1

See full question below

After being pushed away from the departure gate, an aircraft is standing still with the brakes applied and the engine running on very low. When the pilot releases the brakes he finds that the plane still does not start to move. Here are three explanations for why the plane does not move. Which one do you agree with?

A. When a single force that is too weak acts on an object it will not move. When the pilot releases the brakes the only force acting on the plane is that of the

engines but because this force is too weak, the plane will not move.

B. When the pilot releases the brakes two forces act on it. The engines exert a force in one direction and the ground exerts a force on the wheels in the opposite direction. The plane does not move because the force exerted by the ground on the wheels is stronger than the force of the engines.

C. When the pilot releases the brakes two forces act on it. The engines exert a force in one direction and the ground exerts a force on the wheels in the opposite direction. The plane does not move because the force exerted by the ground on the wheels is the same strength as the force of the engines.

A ball thrown vertically upwards at a speed 30mls.
a)Calculate ,how high a ball will travel when its speed is 20m/s?

b)Calculate the maximum height of the ball will reach ?

c)Find time the ball in air ?​

Answers

Ball moving vertically upwards,

initial velocity = 30 m/s

final velocity = 0 m/s [ the ball stops after reaching a certain height ]

Applying v = u + at

               0 = 30 + 10t [ We divide the course of the object into an uphill travel and a descending journey by assuming there is no friction and that g is equal to 10 m/s². ]

Therefore, t = 3 seconds

By symmetry, the descent will take the same amount of time.

Therefore, the object needs a total of 6 seconds before it returns.

using, H=(v²−u²)/2a

ultimate velocity = 0 m/s (it stops when maximum height is reached)

beginning speed = 20 m/s

a = −g = −9.8m/s² (acceleration owing to gravity, negative sign because it is acting downward)

Replace the values

Obtaining, H=20.408m

To know more about vertically upwards velocity, click on the link below:

https://brainly.com/question/28649438

#SPJ9

Suppose the ball is thrown from the same height as in the PRACTICE IT problem at an angle of 29.0° below the horizontal. If it strikes the ground 47.8 m away, find the following. (Hint: For part (a), use the equation for the x-displacement to eliminate v0t from the equation for the y-displacement.)

Answers

The time that the ball took to take off at an angle of  29.0° is  1.13 s


What is an angle?

When two lines meet at a point, they form an angle. The word "angle" refers to the length of the "opening" between these two rays. The symbol is used to denote it.

Radians, a unit of circularity or rotation, and degrees are the two most common units used to measure angles. In daily life, angles are present. For the design of highways, structures, and sports venues, engineers and architects use angles.

The motion along the x-direction is always uniform, so the position along the x-direction is given by

x(t) = (v₀cos∅)t

where,

v₀ is the initial speed,

cos∅ is the angle of launching,

t is the time.

We can re-write the formula as

v₀t = x(t)/cos∅        ...1

Along the y-direction the motion is accelerated (free-fall), so the equation for the vertical position is

y(t) = h = v₀sin∅t - 1/2gt²        ...2

where,

h = 45.0 m is the height of the building

g = 9.8m/s²  is the acceleration of gravity.

Substituting (1) into (2),

y(t) = h = x(t)tan∅ - 1/2gt²

And when this happens, the displacement along the x-axis is x = 47.8 m. Solving the equation for t and substituting the numbers, we find the time of flight:

[tex]t = \sqrt{\frac{2(h-xtan\theta)}{g} }[/tex]

[tex]t = \sqrt{\frac{2(45-47.8tan39^{\circ})}{9.8} }[/tex]

t = 1.13 s

Learn more about angle

https://brainly.com/question/25716982

#SPJ9

Full question

Suppose the ball is thrown from a 45m height  building as in the PRACTICE IT problem at an angle of 29.0° below the horizontal. If it strikes the ground 47.8 m away, find the following. (Hint: For part (a), use the equation for the x-displacement to eliminate v0t from the equation for the y-displacement.)

a) the time of flight s

Can you please explain to me what answer would be correct and how?

Answers

Newton's third law states that; with every action, there is a direct and opposite reaction. As you walk across the flooor, you are applying a force due to your weight on the floor. The floor would apply an an equal force onto your feet which is in opposite direction and this force pushes you forward. This force is called th normal force. Thus, the correct option is

d)


Maria is doing a report on Mathieu Orfila. What fact is she MOST likely to include in her report?
.
AHe solved the first crime that used medical evidence.
BHe studied fingerprints and started to classify them.
CHe was the first person to use bullet comparisons.
DHe explored how poisons affected animals.
??

Answers

Answer:

Its B

Explanation:

Answer:

She is most likely to include that he explored how poisons affected animals.

Explanation:

For a equilibrium shown. There are two strings are strong enough to withstand a maximum tension of 80 n. what is the largest value of w they can support

Answers

The largest value of w (in N)that they can support as shown in equilibrium.

What is equilibrium?

a State of balance between opposing forces or actions that is either static as in nobody acted on by forces whose resultant is zero or dynamic reaction when the rates of reaction in both differentions are equal.

Sol- the total force in x direction is given as,

€Fx=0

T1 sin 37°=T2 sin 53°

100×3/5 =T/2 ×4/5

T2=75 N

The given total force is-

€Fy=0

T1 cos 37°=T2 cos 53°+W

W=80×4/5 -75×3/5

W=19N

Thus

The largest value of W is 19N.

To know more about equilibrium click-

https://brainly.com/question/517289

#SPJ9

If a spring is stretched 4m from its starting length when 20n of force is applied, then how much work (in joules) is done by the spring when it is stretched 10 m?

Answers

ANSWER:

250 J

STEP-BY-STEP EXPLANATION:

F = 20N is required to stretch the spring by 4 meters

We know that the force is equal to:

[tex]F=k\cdot x[/tex]

We solve for k (spring constant):

[tex]k=\frac{F}{x}=\frac{20}{4}=5\text{ N/m}[/tex]

The work done in stretching the spring is given by the following equation (in this case the stretch is 10 meters:

[tex]\begin{gathered} W=\frac{1}{2}k\cdot x^2 \\ \text{ Replacing} \\ W=\frac{1}{2}\cdot5\cdot10^2 \\ W=250\text{ J} \end{gathered}[/tex]

The work required is 250 joules.

The carbon monoxide molecule (CO) consists of a carbon atom and an oxygen atoms separated by a distance of 1.13x10^-10 m, The mass mc of the carbon atom is 0.750 times the mass mo of the oxygen atom, or mc= -0.750mo , Determine the location of the center of the mass of this molecule relative to the carbon atom.

Answers

The location of the centre of the mass of the CO molecule relative to the carbon atom is 6.46 ×10⁻¹¹ m.

What is the centre of mass of the molecule?

The Centre of Mass of that object is a position that is relative to an object or a system of objects. The average position of all parts of the system that are weighted to their masses is known as the Centre of Mass.

Given, the distance between the carbon and oxygen atoms, r = 1.13 ×10⁻¹⁰m.

The ratio of the mass of the carbon and oxygen:

Mc/Mo = 12/16 = 0.750

Consider that Xc is the position of carbon and Xo is the position of the oxygen atom in CO molecule.

The centre of the mass of the system is equal to the:

[tex]X_{co}=\frac{M_CX_C+M_OX_O}{M_C+M_O}[/tex]

[tex]X_{co}=\frac{(M_C/M_O)X_C+X_O}{M_C/M_O+1}[/tex]

As we are calculating the centre of the mass relative to the carbon atom.

Therefore Xc = 0 and Xo= 1.13 ×10⁻¹⁰m.

[tex]X_{CO} =\frac{1.13\times 10^{-10}}{0.750 +1}[/tex]

Xco = 6.46 ×10⁻¹¹ m

Learn more about the centre of mass, here:

https://brainly.com/question/17055505

#SPJ1

The contestant now pulls the block of ice with a rope over his shoulder at 25 degrees. Calculate the minimum force F he must exert to get the block moving. (b) What is its acceleration once it starts to move, if that force is maintained?

Answers

The minimum force that the contestant must exert to get the block moving is 53.33 N and the acceleration of the block is 0.874m/s² once it starts to move and the force is maintained.

When a force is applied at an angle, the force will have two components, one along the floor and one along the vertical.

If the force is F applied at an angle θ, then the vertical component of force is Fsinθ and the floor component is Fcosθ.

In out case θ = 25°,

A.) There will we a frictional force acting on the body which will be opposite to the component of the force which is along the floor, if the floor component just overcomes the static frictional force than the block will start to move.

So, we can write,

Friction force =  Fcosθ

μN = Fcosθ

where,

μ is the coefficient of static friction,

N is the normal reaction by the floor on the block which would be equal to the mg,

F is the minimum force Applied.

Putting all the values,

(0.1)(10)(48) = (F)(cos25°)

48 = F(0.9)

F = 53.33 Newtons.

B.) When the block will start moving,

There will be a kinetic frictional force on it as well as a Fcosθ on it,

The friction will be opposite direction of the motion, Fcosθ will be in the direction of the motion,

So, we can write, The net force on the body

F = Fcosθ - Fr

F = Fcosθ - μN

F = Fcosθ - umgsinθ

When the box is moving the normal reaction N will be equal to the vertical component of the force,

Here, u is the coefficient of kinetic friction,

Putting all the values,

F = 48 - 0.03x10x48x0.42

F = 48 - 6.048

F = 41.95 N

we can also write,

F = ma

where a is the acceleration of the block and m is the mass of the block,

41.95 = 48a

a = 0.874m/s².

To know more about friction, visit,

https://brainly.com/question/4618599

#SPJ9

A contestant in a winter games event pushes a 48 kg block of ice across a frozen lake with a rope over his shoulder at a 25 degree angle.

The coefficient of static friction is 0.1 and the coefficient of kinetic friction is 0.03.

A.) Calculate the minimum force F (in N) he must exert to get the block moving.

B.) What is its acceleration (in m/s^2) once it starts to move, if that force is maintained?

An oscillator is used to create a wave on a string. The frequency of the oscillator is 4 Hz and the waves measures to be 0.5 m apart. What is the speed of the wave

Answers

Given,

Frequency of the wave, f=4 Hz

The wavelength of the wave, λ=0.5 m

The speed of a wave is given by,

[tex]v=f\lambda[/tex]

On substituting the known values in the above equation,

[tex]v=4\times0.5=2\text{ m/s}[/tex]

Therefore the speed of the standing wave produced is 2 m/s

The terminal side contains the point (-6, -8). Find sin θ.Question 1 options:.8.6-.8-.6

Answers

Answer:

The first option: 0.8

Explanation:

Let us draw the angle.

Now,

[tex]\sin \theta=\frac{opposite}{\text{hypotenuse}}[/tex]

Now, using the Pythagoras theorem we find that

[tex]hypotenuse=\sqrt[]{6^2+8^2}[/tex][tex]\begin{gathered} hypotenuse=\sqrt[]{36+64^{}} \\ \Rightarrow hypotenuse=10 \end{gathered}[/tex]

Therefore,

[tex]\sin \theta=\frac{opposite}{hypotenuse}=\frac{opposite}{10}[/tex]

since for our angle, opposite = 8, we have

[tex]\sin \theta=\frac{8}{10}[/tex][tex]\boxed{\sin \theta=0.8.}[/tex]

Therefore, the first choice (0.8) is the correct answer.

why does air pressure decreases with height more rapidly in cold air than in warm air?

Answers

the colder air is more dense, then you would go through more mass (density) in a shorter vertical distance,

Explanation

Consider cold and warm air columns with equal weights, and equal amounts of overlying air.

Colder air is denser because the molecules of air are more closely packed together. As a result, pressure decreases more rapidly as you ascend through cold air than through warm air

It takes a shorter column of cold air to exert the same pressure as a taller column of warm air.

so,Since the colder air is more dense, then you would go through more mass (density) in a shorter vertical distance,

A car of mass m = 1090 kg is traveling down a θ = 11 degree incline. When the car's speed is v0 = 16 m/s, a mechanical failure causes all four of its brakes to lock. The coefficient of kinetic friction between the tires and road is μk = 0.45.
Calculate the distance the car travels down the hill L in meters until it comes to a stop at the end.

Answers

The distance the car of mass 1090 kg travels with a velocity of 16 m / s down the hill having a 11 degree decline until it comes to a stop at the end is 52 m

θ = 11°

m = 1090 kg

g = 9.8 m / s²

μ[tex]_{k}[/tex] = 0.45

W = m g

W = 1090 * 9.8

W = 10682 N

Resolving W into its horizontal and vertical components,

[tex]W_{y}[/tex] = W cos θ

[tex]W_{y}[/tex] = 10682 * cos 11°

[tex]W_{y}[/tex] = 10682 * 0.98

[tex]W_{y}[/tex] = 10468.36 N

[tex]W_{x}[/tex] = W sin θ

[tex]W_{x}[/tex] = 10682 * sin 11°

[tex]W_{x}[/tex] = 10682 * 0.19

[tex]W_{x}[/tex] = 2029.58 N

N = [tex]W_{y}[/tex]

N = 10468.36 N

F[tex]_{k}[/tex] = μ[tex]_{k}[/tex] N

F[tex]_{k}[/tex] = 0.45 * 10468.36

F[tex]_{k}[/tex] = 4710.76 N

[tex]F_{x}[/tex] = m a

[tex]W_{x}[/tex] - F[tex]_{k}[/tex] =  m a

2029.58 - 4710.76 = 1090 * a

a = - 2681.18 / 1090

a = - 2.46 m / s²

Acceleration is negative because the car is decelerating in positive direction.

v² = u² + 2 a s

0 = 16² + ( 2 * - 2.46 * s )

4.92 s = 256

s = 52 m

Therefore, the distance the car travels down the hill is 52 m

To know more about kinetic friction

https://brainly.com/question/17237604

#SPJ1

How many hours would it take a plane to fly 1,500 miles if it was moving at a constant speed of 300 miles/hour. Include units!

Brainliest included

Answers

5 hours
Time= distance / speed
Time= 1,500/ 300
Time= 5

Suppose a 60.0-kg gymnast climbs a rope. (a) What is the tension in the rope if he climbs at a constant speed? (b) What is the tension in the rope if he accelerates upward at a rate of 1.50 m/s2?

Answers

Use the Newton's Second Law of Motion to find the tension in the rope on each case.

a)

Since the gymnast climbs at a constant speed, then the net force acting on the gymnast must be 0. Since there are only two forces acting on the gymnast, namely the tension of the rope and the weight of the gymnast, then:

[tex]\begin{gathered} F_N=T-W=0 \\ \Rightarrow T=W \end{gathered}[/tex]

On the other hand, the weight of the gymnast is equal to the product of its mass times the acceleration of gravity, g=9.81m/s^2:

[tex]\begin{gathered} W=mg \\ =(60\operatorname{kg})(9.8\frac{m}{s^2}) \\ =588N \end{gathered}[/tex]

Then, the tension of the rope is:

[tex]T=588N[/tex]

b)

The forces acting on the gymnast are 2: the tension of the rope and the gravitational pull:

[tex]F_N=T-W[/tex]

From the second law of motion, we know that the net force acting over an object of mass m produces an acceleration a given by:

[tex]F_N=ma[/tex]

Then:

[tex]\begin{gathered} T-W=ma \\ \Rightarrow T=W+ma \end{gathered}[/tex]

Substitute W=588N, m=60kg and a=1.50 m/s^2 to find the tension of the rope:

[tex]\begin{gathered} T=588N+(60\operatorname{kg})(1.5\frac{m}{s^2}) \\ =588N+90N \\ =678N \end{gathered}[/tex]

Therefore, the answers are:

[tex]\begin{gathered} a)T=588N \\ b)T=678N \end{gathered}[/tex]

How did Buddhism and the Maury and contribute to technology in ancient India?

Answers

During the Maurya empire, the Indian lifestyle and manner of existence had been deeply stimulated by Buddhism. Buddhism appealed to human beings of decreased castes because it emphasized individuals' course to enlightenment and salvation, which might be attained in this existence.

The Maurya Empire's political cohesion and inner peace endorsed the expansion of trade in India. at some stage in Ashoka's reign, authorities oversaw the building of essential roadways, and the Mauryan international network of exchange multiplied. India's exports to places like Bactria and Persia blanketed silk, textiles, and spices.

The Maurya Empire become centralized through the conquest of the Indo-Gangetic simple, and its capital town become located at Pataliputra (current Patna). outdoor this imperial middle, the empire's geographical volume became dependent on the loyalty of military commanders who managed the armed towns sprinkling it.

Learn more about the Maurya empire, here

https://brainly.com/question/3106868

#SPJ9

The differences in air pressure that create a sound wave are the result of differences in the velocities of air particles. Is this true or false?

Answers

ANSWER:

true

STEP-BY-STEP EXPLANATION:

The differences in air pressure that create a sound wave are the result of differences in the velocities of the air particles. Due to the Doppler effect, an observer will hear a sound that moves away from her and has a higher pitch. Sound waves will travel faster on a dry day than on a humid day of the same temperature.

Therefore, the statement is true

I can not understand the closed circle and open circle. Please help!

Answers

First, take into account that the open circle means that the point is not included into the domain and the range. The closed point means that such a point is included into both domain and range.

In this case, althoug there is a open circle, there is another part of the curve on which the y value is included into the path.

Then, the range of the graph is:

ran f = [2,7]

Which is equivalent to all number from 2 to 7.

Investigation Question: How do objects get energy?Claim 1: Objects make their own energy.Claim 2: Objects get energy from other objects that have energy.Pick one of the claims above and provide evidence and reasoning for why you believe it is correct.

Answers

Energy can neither be created nor destroyed. It can only be transferred from one object to another.

Claim 2 is correct.

Energy cannot be created or destroyed. So, objects can not make their own energy.

Energy can be transferred or transformed. Energy can be changed in many ways, such as conversion of kinetic energy into potential energy and vice-versa, when one object moves another object, etc.

For example, to lift an object against the gravity, work is done on the object. This work is converted in the form of potential energy, called as gravitational potential energy. When an object, say an apple falls towards Earth from a tree, gravitational potential energy is converted into kinetic energy.

Learn more about energy here,

https://brainly.com/question/5144470.

#SPJ1

A ball is rolled uphill a distance of 5 meters before it slows, stops and begins to roll back. The ball rolls downhill 6 meters before coming to a rest against a tree. What is the ball's displacement?

Answers

We can calculate the displacement using the following formula:

[tex]\begin{gathered} \Delta x=x_f-x_i \\ Where: \\ x_f=Final_{\text{ }}position \\ x_i=Initial_{\text{ }}position \\ so: \\ \Delta x=6m-5m=1m \end{gathered}[/tex]

Answer:

1 m

Anton applies a force on a shopping cart and makes it move forward. What can be said about the forces acting on the shopping cart at the moment Anton applies the force? The forces acting on the shopping cart are...

Answers

Given that, Anton is applying a force on a cart making it move around.

The forces acting on the cart are gravitational force, normal force, frictional force, and the force applied by Anton.

The gravitational force will be acting downwards. From Newton's third law there will be a reaction force to the gravitational force and that is called the normal force. And the formal force will be acting upwards. These two forces are equal to each other in magnitude. And thus they will cancel each other out making the net force on the cart in the vertical direction zero.

The force applied by Anton will be acting forward and the frictional force will be opposing the movement of the cart. The net force in the horizontal direction will be directed forward.

From Newton's second law, the net force is equal to the product of the mass of the cart and the acceleration produced by the net force.

Light hits a mirror at a 45° angle. It will be reflected at an angle _____.

equal to 45°
greater than 45°
less than 45°

Answers

Light hits a mirror at a 45° angle. It will be reflected at an angle equal to 45°

According to law of reflection, when a light ray falls on a surface that is smooth, the angle of reflection will be equal to the angle of incidence. The incident ray, the reflected ray and the normal to the surface all will lie in the same plane.

Reflection is the process in which a light ray gets bounced back after falling on a surface. The angle between in the incident ray and the normal to the surface is known as angle of incidence. The angle between in the reflected ray and the normal to the surface is known as angle of reflection.

Therefore, light hits a mirror at a 45° angle. It will be reflected at an angle equal to 45°

To know more about law of reflection

https://brainly.com/question/15655610

#SPJ1

1. An ordinary light bulb is marked “60A, 120V”, its resistance is ______.

Answers

In order to calculate the resistance, we can divide the voltage and the current, according to the formula below:

[tex]R=\frac{V}{I}[/tex]

Where R is the resistance (in ohm), V is the voltage (in Volt) and I is the current (in Ampere).

So, using V = 120 V and I = 60 A, we have:

[tex]R=\frac{120}{60}=2\text{ ohms}[/tex]

Therefore the resistance is 2 ohms.

Best player in nba right now

Answers

Best player in NBA right now Giannis Antetokounmpo.

The National Basketball Association, NBA is a professional basketball league in North America. The league consists of 30 teams and is one of the predominant expert sports activities leagues within the united states of America and Canada. It's far the finest guys professional basketball league within the international.

To sell great aware device of technical education where excellence, relevance to market desires and participation with the aid of all stakeholders are prime and principal determinants.

The NBA is a 70-year-vintage employer born from innovation. It turned into June 1946 in the big apple city while Boston garden owner Walter Brown found out that primary ice hockey arenas, which were vacant maximum nights, will be used to host basketball games.

Learn more about players of NBA here:- https://brainly.com/question/26995044

#SPJ1

A cannon ball is shot at 10 m/s at an angle of 30 degrees. It takes 2.2 seconds for the ball to reach the ground. (A) How high is the cliff that the cannonball is shot from? (B) How far from the base of the cliff does the ball land?

Answers

A ) The height of cliff from which the cannonball is shot from = 12.72 m

B ) The distance from the base of the cliff = 19.14 m

- H = [tex]u_{y}[/tex] T - 1 / 2 g T²

T = Total time taken

g = Acceleration due to gravity

H = Height above the ground  

[tex]u_{y}[/tex] = Y-component of initial velocity

T = 2.2 s

g = 9.8 m / s²

θ = 30°

u = 10 m / s

[tex]u_{y}[/tex] = u sin θ

[tex]u_{y}[/tex] = 10 * sin 30°

[tex]u_{y}[/tex] = 5 m / s

- H = ( 5 * 2.2 ) - ( 0.5 * 9.8 * 2.2 * 2.2 )

- H = 11 - 23.72

H = 12.72 m

R = [tex]u_{x}[/tex] T

[tex]u_{x}[/tex] = u cos θ

[tex]u_{x}[/tex] = 10 * cos 30°

[tex]u_{x}[/tex] = 8.7 m / s

R = 8.7 * 2.2

R = 19.14 m

Therefore,

A ) The height of cliff from which the cannonball is shot from = 12.72 m

B ) The distance from the base of the cliff = 19.14 m

To know more about Projectile launched from a height

brainly.com/question/14467028

#SPJ1

Other Questions
In Book I Penelope shows her greatest fidelity to Ulysses by and shows possible disloyalty by . What does the leading term of 5x^4+4x^36x^2+8 tell you? How do you think the immigrants were feeling as they were processed through ellis island? what thoughts do you think might have gone through their heads? What is a planet's orbit? Find the area of the rectangle.1 1/2 yd58/ 8 yd A clone has a volume of 1, 526.81 mm^3 and a height of 19mm. Find the radius of the cone geometry pls help its easy im just slow The Dust Bowl (1930 - 1936)4. How many dust storms happened in 1933? How long did the storm in April 1933 last? I need questions 10 , 13 , 15 , and 16 solved with steps and graph let's consider that ethyl acetate is the ideal solvent for this tlc experiment. what might have happened to the spots if you had used t-butyl methyl ether as the eluent instead of ethyl acetate. QUESTION 9 1 POINTSimplify: ( 12 + 1/2 m) 0, where + #0.4 2If the given expression is undefined enter as your answer.Provide your answer below:Content attribution A charge of 15C has flowed past a point in the circuit with a current of 2A. How long has this charge been flowing through this circuit?Round the answer to 2 decimal places. How many degrees mustFigure A be rotatedcounterclockwise aroundthe origin in order to lineup with Figure B?BA. 90B. 180C. 270D. 360 PLEASE HELP 50 POINTS + BRAINLY FOR ALL AND CORRECT ANSWERS!!! Read this excerpt from a policy document that informs employees about payroll procedures:A) Each employee will submit a completed time sheet that shows the number of hours worked, sick time, and any vacation time taken. B) Managers must review each time sheet for accuracy and then sign it. The project director will then approve and sign each time sheet. C) If the company cant directly deposit pay into employees bank accounts, the company will distribute payroll checks by 5 p.m. on the first Monday after the two-week pay period has ended. (D) The company will deduct taxes and benefits from pay.In which scenario would the company need to refer to the policy document?A.An employee wants to apply for vacation time but hasnt submitted the time sheet to the manager.B.An employee wants a paycheck immediately after finding out that his bank account wont accept direct deposit payments.C.An employee has submitted the time sheet to her manager but hasnt been notified of its approval.D.An employee demands a paycheck even though his manager hasnt approved the time sheet.E.An employee demands a paycheck even though she hasnt applied for sick time. 100 POINTS + BRAINLEIST IF CORRECT The holiday _____ fantastic! We had a great time. The Cold War: Causes & Alliances: Communism is based on the virtues of_________________ and ________ preschool suffix............................. Write a properly formatted hypothesis statement to answer this question: how does the amount of dissolved salt affect the boiling point of water? specify how you plan to change the independent variable by using terms such as increase or decrease. also, specify how the dependent variable will change in response by using terms such as increase, decrease, or stays the same.